Which one of the following, if true, would cast doubt on the argument in passage B but bolster the argument in passag...

Alexandra on July 19, 2020

Explanation

Can this be explained please

Replies
Create a free account to read and take part in forum discussions.

Already have an account? log in

Shunhe on July 20, 2020

Hi @alliehall21,

Thanks for the question! So in this question we’re looking for something that’ll make passage B worse off but will bolster passage A’s argument. And in general, we should have a basic idea of what the two passages think. Passage A thinks that purple loosestrife is bad for native species, whereas passage B thinks that purple loosestrife isn’t as bad as some people make it out to be.

Now let’s take a look at (A). Assume it’s true that localized population reduction is often a precursor to widespread endangerment of a species. Well, this bolsters passage A’s argument, since passage A talks about how there’s been localized population reduction as a result of purple loosestrife, and passage A is also arguing overall that purple loosestrife is super bad. And so if purple loosestrife leads to localized population reduction, that might be a sign that it’s leading to widespread endangerment of species, which would be super bad. And so (A) definitely bolsters passage A’s argument.

Now does it cast doubt on passage B’s argument? Why yes, it does. Remember, passage B is trying to say that purple loosestrife isn’t that big of a deal. But passage B acknowledges that there has been some localized population reduction due to purple loosestrife. So if answer choice (A) is true, then that makes it less likely that purple loosestrife isn’t as dangerous as it seems, since it could be leading to widespread endangerment. And so that casts doubt on passage B’s claim.

We can also see that (B) and (E) are irrelevant, (D) doesn’t help out passage A, and (C) actually helps passage B (which is the opposite of what we want). So all of those are wrong answers.

Hope this helps! Feel free to ask any other questions that you might have.

Mateen on April 21, 2022

Can I get an explanation as to why (C) actually helps passage B? I am having trouble understanding how it bolsters the argument presented in B. Thanks in advance!

Emil on April 26, 2022

Hi Mak16153,

Passage B argues that the anti-loosestrife lobby is motivated by profit, that is, people only want the plant gone because it has an adverse economic impact on the areas in which it grows. C tells us that the loosestrife threatens species that are economically important, and that the loss of these species (due to the loosestrife) would have a large impact on their revenues. This supports the contention in passage B by giving an additional fact to support the idea that opponents of the loosestrife are motivated by profit, it shows that the loosestrife would drive down profits.